LSAT and Law School Admissions Forum

Get expert LSAT preparation and law school admissions advice from PowerScore Test Preparation.

 tperez1
  • Posts: 2
  • Joined: Sep 19, 2014
|
#16763
This is probably flying over my head but I'm confused as to why the answer is C (fails to take into account the possibility that patients at Edgewater Hospital tend to be treated for different illnesses than patients at University Hospital) is correct when the stimulus premise indicates that the patients at both hospitals are treated with similar illnesses?

Thank you,
Tiffany
 Nicholas Bruno
PowerScore Staff
  • PowerScore Staff
  • Posts: 62
  • Joined: Sep 27, 2011
|
#16768
Hi!

I think maybe it is a simple misreading of the stimulus that is confusing you! The stimulus does not say that the hospitals treat similar illnesses. It says that *recovery rates* (i.e. the percentage of times the patients are successfully cured) for similar illnesses are the same.

Does that answer your concern? It might just be a simple misreading!

Nick
 tperez1
  • Posts: 2
  • Joined: Sep 19, 2014
|
#16992
It does! Thanks so much!

Tiffany
 jcough346
  • Posts: 35
  • Joined: Aug 05, 2016
|
#30720
Is the key to this question the scope shift from "average length of stay for patients" to "patients with similar illnesses"?
 Adam Tyson
PowerScore Staff
  • PowerScore Staff
  • Posts: 5191
  • Joined: Apr 14, 2011
|
#30790
I'd say that's accurate, jcough. Another way to put it as that the author assumed, without warrant, that the patients at the two hospitals had similar illnesses.
 srcline@noctrl.edu
  • Posts: 243
  • Joined: Oct 16, 2015
|
#32779
Hello,

Again, missed this Flaw question. So I read on another thread that you're trying to find the break in the argument. To me the break was that the conclusion is equating that Univ. Hosp. could decrease its average length of stay without affecting quality of care. So I eliminated B, D, E. I distinctly remember reading the LR bibles that we are solely concerned with the flaw in the reasoning of the argument..or in LSAT terms the CONCLUSION. So this a an example of a False dilemma

How is C relevant at all....."treated for different illnesses" is not an element that is discussed in the conclusion. Why do we care about this when the author is equating average length with quality of care. What if Univ. Hops has better conditions than Edgewater? Wouldnt that also be a possibility of decreasing avg. length of stay?

(another question, about question 4 in this same section, was removed and used to create another thread - lsat/viewtopic.php?t=13398)

I apologize for the long post.....Thankyou for your explanations
Sarah
 Adam Tyson
PowerScore Staff
  • PowerScore Staff
  • Posts: 5191
  • Joined: Apr 14, 2011
|
#32784
Take another look at the stimulus, Sarah, and you'll see that "patients with similar illnesses" is an element. So, if you are treated for pneumonia at Edgewater, your recovery rate should be the same for a patient treated for the same type of pneumonia at University. A patient treated at University for diverticulitis should recover at the same rate as a diverticulitis patient at Edgewater.

Despite these similarities, patients at University are, on average, staying in the hospital longer than patients at Edgewater. The author then concludes that the University patients could probably go home sooner than they have been. He thinks they would be just fine being sent home earlier.

There are a few assumptions going on here, one of which is that Edgewater could not improve quality of care by allowing patients to stay longer. Maybe the recovery rates are the same, but Edgewater is sending patients home before they are fully recovered? Maybe there are factors other than recovery that warrant staying longer?

The big, obvious, numbers assumption, though, is the one at issue in answer C. We know that recovery rates are the same at the two hospitals for similar illnesses, but we also know that patients stay longer at University. Could it be that the patients at University are being treated for different illnesses than the patients at Edgewater, and that those different illnesses have longer recovery rates? Edgewater may treat a lot of diverticulitis, with a recovery rate of four days, while University treats more pneumonia, with a six day recovery rate? The author assumed that the two hospitals are actually treating patients with similar illnesses.

I'm not sure I would call this a False Dilemma, Sarah. I would say it is a problem based solely on a bad assumption about numbers.

For the answer to your other question, I created a new thread here:

lsat/viewtopic.php?t=13398
 taylorballou
  • Posts: 18
  • Joined: Feb 18, 2017
|
#34262
Hi,

Can you please provide clarification on the reason for eliminating answer choice D? I narrowed down the answer choices to C and D, and then ended up guessing D as I wasn't sure which answer was stronger than the other. I was thinking that D is correct because some illnesses require a longer hospital stay in order for the patient to recover, so the argument ignored this aspect of the recovery rate by saying both hospitals should have the same length of stay. Now that I am re-reading the answer choices, I suppose C covers what's discussed in answer D as well because a hospital treating different types of illnesses would imply that there is sometimes a difference in recovery rate?

Thanks,

Taylor
 Ricky_Hutchens
PowerScore Staff
  • PowerScore Staff
  • Posts: 59
  • Joined: Oct 12, 2015
|
#34275
Hi Taylor,

I think you are misreading D because your explanation for D is why C is correct. D actually says that the argument ignores the possibility that how long a patient stays in the hospital might be relevant to his recovery. However, the argument doesn't do this because it is comparing recovery rates for similar patients with similar illnesses. But the stimulus does ignore the possibility that the hospitals might be treating vastly different patients. And this would explain the difference in their numbers.
 biskam
  • Posts: 124
  • Joined: Aug 18, 2017
|
#41924
can someone please tell me why D is incorrect? having trouble understanding the aforementioned explanation.

Get the most out of your LSAT Prep Plus subscription.

Analyze and track your performance with our Testing and Analytics Package.